1answer.
Ask question
Login Signup
Ask question
All categories
  • English
  • Mathematics
  • Social Studies
  • Business
  • History
  • Health
  • Geography
  • Biology
  • Physics
  • Chemistry
  • Computers and Technology
  • Arts
  • World Languages
  • Spanish
  • French
  • German
  • Advanced Placement (AP)
  • SAT
  • Medicine
  • Law
  • Engineering
Nikitich [7]
4 years ago
9

Rewrite as a simplified fraction3.83 =​

Mathematics
2 answers:
alukav5142 [94]4 years ago
7 0

Answer:

3 83/100

Step-by-step explanation:

The part 83/100 can not be simplified more because 83/100 is in its simplest form already.

sveticcg [70]4 years ago
3 0

Answer:

3 83/100

Step-by-step explanation:

Hello Applebannana11!

Let's take a slower more simple approach first.

Since the places for decimals are ten(ths) and hundred(ths), we can see that it is going to be \frac{83}{100}.

Since, 3 is our whole digit, it will be the first thing.

3 \frac{83}{100}.

Now, we check if \frac{83}{100} can be simplified.

We see the GCF of 83 and 100 is 1.

Therefore, \frac{83}{100} cannot be simplified.

Since we don't need to simplify 3, our final answer is $\boxed{3 \frac{83}{100} }.

You might be interested in
1. if 50 people pay $50 a month for their cellphone bill how much money is it?
Murljashka [212]
1) 50*50, our 2500
2) 1500
3)1500
4) 900

just multiply t the number of people by the amount each pays
5 0
3 years ago
Triangle A B C is congruent to triangle E F G.
Elan Coil [88]
Angle CBA is congruent to angle GEF
3 0
4 years ago
Read 2 more answers
Write an expression that evaluates to true if the value of the integer variable number of prizes is divisible (with no remainder
natta225 [31]

Answer:

A=BQ

Step-by-step explanation:

In order to find an expression, you can use the definition of the dividend in a division:

A=BC + R

where A is the dividend, B is the divisor, Q is the quotient (the result of the division) and R is the remainder of the division.

Let A represent the integer variable number of prizes and B represent the integer variable number of participants.

In this case R=0 and B≠0, therefore:

A=BQ

A is divisible by B if A can be written as an integer multiple of B. In other words, you have to find an integer number Q that multiplied by B produces A.

8 0
3 years ago
30 \div \frac{1}{3} 30÷ 3 1 ​
ioda

Answer:

<h2>90</h2>

Step-by-step explanation:

30 \div \frac{1}{3} \\\\\mathrm{Convert\:element\:to\:fraction}:\quad \:30=\frac{30}{1}\\=\frac{30}{1}\div \frac{1}{3}\\\\\mathrm{Apply\:the\:fraction\:rule}:\quad \frac{a}{b}\div \frac{c}{d}=\frac{a}{b}\times \frac{d}{c}\\=\frac{30}{1}\times \frac{3}{1}\\\\\mathrm{Apply\:rule}\:\frac{a}{1}=a\\=30\times \:3\\\\=90

7 0
3 years ago
Read 2 more answers
Y=1/2x+2 need answers on graph
Aneli [31]

Answer:

the slope is 1/2 and the Y - intercept is 2

X       Y

0       2

2       3

Step-by-step explanation:

5 0
4 years ago
Other questions:
  • On a math test, the students are asked to
    11·1 answer
  • Write x +y = 3 in function notation.
    7·2 answers
  • HELP!!!! 9 through 12
    6·1 answer
  • The cost of a 24 pack(12 ounce cans) of coca cola is 7.68.The cost of a 12 pack(12 ounce cans) of Pepsi cola is 4.98
    11·1 answer
  • Which list shows the triangles in order from greatest area to least area
    15·1 answer
  • I bought a box of 124 crayons for the art class to share. The crayons cost $21.08 with tax included. What was the cost per crayo
    11·2 answers
  • A data set consists of the following data points:
    15·1 answer
  • Evaluate the expression for p = 3.<br><br> 7p
    9·2 answers
  • Pls whoever knows how to do this best answer smart people 100% this geometry work give Brainly pls help
    13·1 answer
  • 46) write the expression: The quotient of p and q is twelve less than three times the sum of P and q
    14·1 answer
Add answer
Login
Not registered? Fast signup
Signup
Login Signup
Ask question!